×

UPSC Courses

UPSC Previous Year Solved Papers

CURRENTAFFAIRS

1. Consider the following organisations:
  1. International Bank for Reconstruction andDevelopment
  2. International Finance Corporation
  3. International Fund for Agricultural Development
  4. International Monetary Fund

Which of these are agencies of the United Nations?

Explanation: D. All the Agencies given in the question are the part of the United Nations.

2. Match List-I with List-II and select the correct answer using the codes given below the lists:

Explanation: B. "The Struggle in My Life" is written by Nelson Mandela, " The Struggle and the Triump" is written by Lech Walsesa, "Friends and Foes" is written by Sheikh Mujibur Rahman and "Rebirth" is written by Leonid Brezhenev, the former president of U.S.S.R

3. Falun Gong is:

Explanation: D. Falun Gong is a spiritual movement in China against which the Chinese authorities have taken stren action.

4. Who is the scientist in whose honour the "Chandra" X-ray telescope has been named?

Explanation: D. "Chandra" X-ray telescope has been named after famous physicist of Indian origin Subramanyam Chandrasekhar.

5. Match List-I with List-II and select the correct answer using the codes given below the lists:

Explanation: D. World Environment Day

6. The earlier name of the WTO was:

Explanation: B. GATT

7. The location of the space organisation units have been marked in the given map as 1, 2, 3 and 4. Match these units with the list given below and select the correct answer using the codes given below the lists:
  1. I.S.R.O
  2. I.I.R.S.
  3. N.R.S.A
  4. S.A.C.

Codes:

Explanation: D. ISRO is situated in Banglore, which is marked as '4', Indian Institute of Remote Sensing (IIRS) is located in Dehradun which is marked as '1', National Remote Sensing Agency (NRSA) is situated in Hyderabad, marked as '3' and Space Application Centre (SAC) is situated in Ahmedabad which is marked as '2' in the map.

8. The largest number of Buddhist is found in:

Explanation: C. Maharashtra is the state where 6.39% of population is Buddhist. They are 5.84 million Buddhists found in Maharashtra according to 2001 census. It is 73.5% of total Buddhist population.

9. Match List-I with List-II and select the correct answer using the codes given below the lists:
  1. A. Central Institute of medicinal and Aromatic Plants 1. Chandigarh
  2. B. Centre for DNA Finger Printing and Diagnostics 2. Hyderabad
  3. C. Institute of Microbial Technology 3. New Delhi
  4. D. National Institute of Immunology 4. Lucknow

Codes:

Explanation: B. The central Institute of Medicinal and Aromatic plants is located in Lucknow, Uttar Pradesh center for DNA finger printing and diagnosis is located in Hyderabed of Andhra pradesh, Institute of Microbial technology is situated in Chandigrah and National Institute of Immunology is located in New Delhi.

10. Consider the following statements about the minorities India:
  1. The Government of India has notified five communities, namely, Muslims, Sikhs, Christians, Buddhists and Zoroastrians as Minorities
  2. The National Commission for Minorities was given statutory status in 1993
  3. The smallest religious minority in India are the Zoroastrians
  4. The Constitution of India recognizes and protects religious and linguistic minorities

Which of these statements are correct?

Explanation: B. As per current data, The following communities have been notified as minority communities by the Government of India, Ministry of Minority Affairs; Sikhs, Muslims, Christians, Zoroastrians, Buddhists and Jains. The first statutory National Commission was set up on 17th May 1993. Jains form the smallest religious minority in India.

11. Consider the following organizations:
  1. Atomic Minerals Directorate for Research and Exploration
  2. Heavy Water Board
  3. Indian Rare Earths Limited
  4. Uranium Corporation of India

Which of these is/are under the Department of Atomic Energy?

Explanation: D. All the four organisations are under the department of Atomic energy.

12. Consider the following statements regarding the armed forces:
  1. First batch of women pilots was commissioned in Indian Air Force in 1996
  2. Officers' Training Academy is located in Nagpur
  3. Southern Command of Indian Navy has it headquarters at Chennai.
  4. One of the Regional Headquarters of Coast Guard is located at Port Blair

Which of these statements are correct?

Explanation: D. Officer's Training Academy (OTA) is located in Chennai and Gaya. The Southern command of Indian navy has its headquarters at cochin not in Chennai. The Regional Headquarters of coast Guard are located at Mumbai, Chennai and Port Blair. The First batch of Women Pilots was commissioned in Indian Force in 1994.

13. Which of the following committees examined and suggested Financial Sector reforms?

Explanation: D. Narasimhan committee was constituted on financial sector reforms in 1991 and on banking sector reforms in 1998.

14. Which one of the following statements is correct?

Explanation: C. The United Nation took over East Timor in 1999 to prepare for its independence and East Timor finally got independence on 20 may 2002.

15. In which one of the following countries is rupee its currency?

Explanation: D. The rupee is the currency of Seyhelles. It is locally called roupi, it is subdivided into 100 cents. Currency of Bhutan is Ngultrum, Currency of Malaysia is Malaysian Ringget and Currency of Maldives is Maldives Rufiyaa.

16. Which one among the following languages has largest number of speakers in the world?

Explanation: A. According to 2005 edition of the National encyklopedin, the largest number of speakers in the world are, Portuguese (213 millikon speakers); Bengali (215 million speakers); Japanese; (127 million speakers); French (130 million speakers).

17. Which among the following countries was the earliest to give women the right to vote?

Explanation: C. New Zealand gave the right to vote to women in 1893.

18. With reference to Indian defence. Which one of following statements is NOT correct?

Explanation: A. Correct option is A.

19. Match List I with List II and select the correct answer:

Explanation: C. Brijmohan Lal Munjal is chief of Hero Honda i.e.,he is related to Automobile Industry. Kiran Karnik is chief of NASCOM i.e., he is related to Software Industry. Kiran Muzumdar Shaw is related to biotechnology and Sunil Mittal is chief of Bharti Group i.e., related to Telecom industry.

20. Which one of the following statements is NOT correct?

Explanation: A. National Song was composed originally in Sanskrit (not in Bengali).

21. The thermal power plant of Bokaro is located in:

Explanation: C. Bokaro thermal power plant is located in Jhadkhand, near Konai Dam

22. Which one among the following has the largest shipyard in India?

Explanation: B. Kochi shipyard is the largest shipyard in India.

23. Dilip Kaur Tiwana is well known as a:

Explanation: A. Dr. Dalip Kaur Tiwana is a foremost novelist and short-story writer of contemporary Punjabi literature.

24. Consider the following statements regarding the relations between India and Pakistan:
  1. During Shimla Agreement, Indira Gandhi ,and Zulfikar Bhutto agreed to maintain the sanctity of LOC
  2. Lahore summit took place in the year 1997
  3. Islamabad summit was held between Rajiv Gandhi and Nawaz Sharif

Which of these statements is/are correct?

Explanation: D. The Lahore Declaration is a bilateral agreement and governance treaty between India and Pakistan. The treaty was signed on February 21, 1999, at the conclusion of a historic summit in Lahore and ratified by the parliaments of both countries the same year. Islamabad Summit was held between Rajiv Gandhi and Benazir Bhutto.

25. Consider the following international language:
  1. Arabic
  2. French
  3. Spanish

The correct sequence of the language given above in the decreasing order of the number of their speakers is:

Explanation: A. Total speaker of Spanish in world is 329 million, Arabic speakers are 220 million and French speakers are 200 million. So, the correct sequence in decreasing order of the number of speakers are Spanish, Arabic and French.

26. Match List-I with List-II and select the correct answer using the codes given below:
  1. A. United Nations Development Programme (UNDP) 1. Nairobi
  2. B. United Nations Environment Programme (UNEP) 2. Vienna
  3. C. United Nations Industrial DevelopmentOrganization (UNIDO) 3. Berne
  4. Postal Union (UPU) 4. New York

Codes:

Explanation: B. The headquarters of United Nations Development Programme is in New york, the headquarters of United National Environment Programme is in Nairobi, The headquaters of United National Industrial Development Organisation is present in Vienna and Headquarters of Universal Postal Union is in Beru.

27. World's longest ruling of government is from:

Explanation: B. World's longest ruling government is of Fidel Castro of Cuba. Fidel Castro came in power in 1959 following the Cuban revolution and became Prime Minister of Cuba in 1961 and from 1976 he became the President of Cuba till date.

28. Match List-I (State/Province/Overseas Territory) with List-II (Country) and select the correct answer using the codes given below:

Explanation: B. British Columbia is western most provinces of Canada, Bavaria is a state of Germany. Gibralter is the overseas territory of UK and Rhoda Island is the smallest state in USA.

29. Match List-I (New names of the Countries) with List-ll (Old names of the Countries) and select the correct answer using the codes given below:

Explanation: B. Benin

30. Consider the following statements:
  1. Montenegro and Serbia agreed to a new structure for the Yugoslav Federation
  2. Croatin remained under the Hungarian Administration until the end of First World War
  3. Claims to Macedonia Territory have long been a source of contention between Belgium and Greece

Which of the statements given above are correct?

Explanation: B. All the statements except '4' are correct. Slovenia was declared independent from Yugoslavia.

31. Match List-I (persons) with List-II (Positions) and select the correct answer using the codes given below:
  1. A. B.P. Mishra 1. Executive Director, IMF
  2. B. Suresh Kalamadi 2. Chairman, Steel Authority of India Limited
  3. C. Praful Patel 3. President Indian Olympic Association
  4. D. V. S. Jain 4. Vice-President, South Asian Region, World Bank

Codes:

Explanation: B. B.P. Mishra is the Executive Director of IMF, Suresh Kalmadi is the president of Indian Olympic Association, Praful Patel is the Vice President, South Asian Region, World Bank and V.S. Jain is the chairman of Steel Authority of India Limited.

32. Who among the following is well known as an exponent of flute?

Explanation: C. Ronu Mazumdar is well known exponent of flute whereas Debu Chaudhuri is an exponent of sitar and Sahafaat Ahmad is an exponent of tabla.

33. Which one of the following statements is not correct?

Explanation: A. The largest Buddhist Monastry is in Tawang in Arunachal Pradesh not in Assam

34. Consider the following statements:
  1. The Islamic Calendar is twelve days shorter than the Gregorian Calendar.
  2. The Islamic Calendar began in AD 632
  3. The Gregorian Calendar is a lunar calendar

Which of the statements given above is/are correct?

Explanation: There is no any true option.

35. Match List-I (Institute) with List-II (Located At) and select the correct answer using the codes given below:
  1. A. Indian Institute of Geomagnetism 1. Coimbatore
  2. B. International Advanced Research Centre for Powder Metallurgy and New Materials 2. Mumbai
  3. C. Salim Ali Centre for Ornithology and Natural History 3. Jabalpur
  4. D. Tropical Forestry Research Institute 4. Hyderabad

Codes:

Explanation: C. A-2; B-4; C-1; D-3.

36. Which one of the following pairs is not correctly matched?

Explanation: B. Central Organisation for Railway Electrification is present in Allahabad, Ambala, Bhubneshwar, Chennai, Banglore, Secunderabad, Lucknow, Kota and Gorakhpur not in Varanasi.

37. Who among the following was never the Lok Sabha Speaker?

Explanation: A. Hukum Singh (April 1962-March 1967), GS Dhillon (April 1969-Dec 1975), Baliram Bhagat (Jan 1976-March 1977)

38. Which of the following pairs is correctly matched?
  1. Department of Women and Child Development - 1. Ministry of Health and Family Welfare
  2. Department of Official Language - 2. Ministry of Human Resource Development
  3. Department of Drinking Water Supply - Ministry of Water Resources

Select the correct answer using the codes given below:

Explanation: C. The Department of Women and Child Development was set up in the year 1985 as a part of the Ministry of Human Resource Development. With effect from 30 January 2006, the Department has been upgraded to a Ministry. Department of official language comes under Ministry of Home Affairs.

39. Consider the following statements:
  1. P.V. Narasimha Rao's government established diplomatic relations between India and Isreal
  2. Ariel Sharon is the second Prime Minister of Isreal to have visited India

Which of the statements give above is/are correct?

Explanation: A. The diplomatic relation between India and Isreal was established in 1992 under the Narsimha Rao regime. Ariel Sharon happened to be the first Prime Minister of Israel to visit India.

40. Consider the following companies:
  1. Voltas
  2. Titan Industries
  3. Rallis India
  4. Indian Hotels

Which of the above companies are in the Tata Group of industries?

Explanation: D. Voltas, Titan Industries, Rallis India and Indian Hotels are all companies under Tata Group.

41. Tarun Bharat Sangh, an organisation based in a village near Alwar (Rajasthan) has become famous because of:

Explanation: D. Rain water harvesting.

42. Famous Golf player Vijay Singh is from which one of the following countries?

Explanation: A. Famous Golf player Vijay Singh is from Fiji.

43. Which of the following cricketers holds the record for the highest score in a Cricket test match innings by an Indian?

Explanation: D. Among the given options, Highest score in a cricket match innings by an Indian is 281 runs of V.V.S. Laxman.

44. Match List-I (Distinguished Ladies) with List-II (Area of work) and select the correct answer using the codes given below:

Explanation: B. Jhumpa Lahiri is related to novel writing, Sunita Narain is related to science and environment, Naina Lal Kidwai is related to banking and Ravina Raj Kohli is related to television media.

45. Match List-I (Sports-person) with List-II (Sport/Game) and select the correct answer using the codes given below:

Explanation: D. Shikha Tandon is associated with Swimming, Ignace Tirkey is asociated with Hockey, Pankaj Advani is associated with Snooker and Rohan Bopanne is associated with Lawn Tennis.

46. Match List I (Drugs / Chemicals) with List II (Their uses) and select the correct answer using the codes given below the Lists:
  1. A. Atropine 1. Local anaesthesia
  2. B. Ether 2. Heart trouble
  3. C. Nitroglycerine 3. Dilation of pupil
  4. D. Pyrethrin 4. Mosquito control

Codes:

Explanation: D. Ether is used as an anesthesia at the time of surgery to prevent pain. Atropine is used as eye drops to dilate the pupil before eye examination. Nitroglycerine is widely used in the treatment of heart. Pyrethrin is biodegradable, non-persistent pesticide which is used to kill mosquitoes, fleas and other insects.

47. Consider the following statements:
  1. The Headquarters of the International Organisation for standardization are located in Rome.
  2. ISO 9000 relates to the quality management system and standards.
  3. ISO 14000 relates to environmental management system standards.

Which of the statements given above is/are correct?

Explanation: C. The HQ of ISO is Geneva, Switzerland.

48. Which one of the following is not an ASEAN member?

Explanation: B. China is not a member of ASEAN. It is a part of ASEAN + 3.

49. Consider the following statements:
  1. The Constitution of the United States of America came into force in year 1810.
  2. All revenue bills must originate in the House of Representative of the US Congress.
  3. George W. Bush is the only President in the history of the United States of America. Whose father was also the President of the United States of America.

Which of the statement given above is/are correct?

Explanation: B. The Constitution of USA came in to force in 1787. In the history of USA, John Adams and John Quincy Adams, and George H.W. Bush and George W. Bush are the two father - son duos who have been the President of USA.

50. Consider the following statements:
  1. The Parliament of Russia is called Federal Assembly
  2. The Council of the Federation in the Russian Parliament is the lower house.
  3. The name of the upper house in the Russian Parliament is state Duma.

Which of the statements given above is/are correct?

Explanation: D. Only statement (1) is correct as statements 2 and 3 are mismatched. Federation Council is Upper House of Russian Parliament and State Duma is the Lower House. Both are located in Moscow.

51. Match items in the List-I (country) with those in the List-II (Name of Parliament) and select the correct answer using the codes given below the lists:
  1. A. Netherlands 1. Diet
  2. B. Ukraine 2. States General
  3. C. Poland 3. Supreme Council
  4. D. Japan 4. Sejm

Codes:

Explanation: B. States General is the lower house of the bicameral parliament of the Netherlands. The Supreme council i.e. Verkhovna Rada is the Parliament of Ukraine. The Sejm of republic of Poland is the lower house of the Polish Parliament. The National Diet is Japan

52. Consider the following statements:
  1. The man Booker prize is awarded to citizen of any of the countries of the British common wealth or the Republic of Ireland.
  2. A leading London based software company underwrites the man Booker Prize presently.
  3. The winner of the man Booker Prize in year 2004 is a South Asian.

Which of the statement given above is/are correct?

Explanation: A. The Man Booker Prize is a literary Prize awarded each year for the best original novel written in the English language and published in UK from its inception, only novels written by commonwealth, Irish and South African citizens were eligible to receive the prize; in 2014, however, this eligibility was widened to any English language novel. The winner of the man Booker Prize in year 2004 was Alan Hollinghurst who belonged to United Kingdom. The Man Booker Prize is awarded by Man Group.

53. Consider the following statements:
  1. The Charter of the United Nations Organization was adopted at Geneva, Switzerland in June, 1945.
  2. India was admitted to the United Nations Organization in the year 1945.
  3. The Trusteeship Council of the United Nations Organization was established to manage the affairs of territories detached from Japan Italy after the second World War or such territories not under the control of a country at that time.

Which of the statements given above is are correct? .

Explanation: C. Statement (1) is not correct. The charter of UNO was adopted at San Francisco in June 1945.

54. Which one of the following pairs is not correctly matched?

Explanation: C. Ghana and Dutch Guiana are not related to each other.

55. In which country is Bandung, where the conference of African and Asian nations was held which led to establishing Non-Aligned Movement (NAM); situated?

Explanation: C. In Indonesia the conference of African and Asian nations was held which established to Non-aligned movement.

56. In which one of the following countries, is Tamil a major language?

Explanation: D. Tamil is the major language in Singapore

57. capital of the given country (Country given in the brackets)?

Explanation: B. Karachi was the capital of Pakistan from 1947

58. Which one of the following pairs is not correctly matched?

Explanation: A. Seikan Rail Tunnel is world's longest tunnel with length of 34 miles is located in Japan. Seikan railway tunnel is Japan's 53.85 km long tunnel beneath the Tsugaru Strait and connects the Aomori Prefecture on Honshu Island and the Hokkaido Island. With its track located 140m below the seabed, the Seikan tunnel is the world's deepest and longest railway tunnel.

59. For which one of the following countries is Spanish not an official language?

Explanation: C. French is the official language of Republic of Congo, not Spanish. But spanish is the official language of chile, Columbia nad Cuba.

60. What was Komagata Maru?

Explanation: C. Komagata Maru was a Japanese ship carrying passengers from Singapore to Vancouver. The ship was anchored at Calcutta in September 1914.

61. Which one of the following pairs is not correctly matched?

Explanation: C. John Major Conservative Party

62. Which one of the following cities is the seat of Parliament of South Africa?

Explanation: D. Since, 1910 as the seat of Parliament, Cape Town has been the legislative capital of South Africa, Pretoria the Administrative capital and Bloemfontein the Judicial Capital.

63. Which one of the following pairs is not correctly matched?

Explanation: C. Armoured corps centre and school is located at Ahmednagar, Maharashtra.

64. For which one of the following, is Satara well known?

Explanation: A. Satara is well known for Thermal Power Plant.

65. Consider the following statements.
  1. The number of post office in India is in excess of 1 lakh
  2. Bharat Sanchar Nigam Limited (BSNL) was formed in the year 1997.
  3. Telecom Regulatory Authority of India (TRAI) was established in the year 2000.

Which of the statements given above is/are correct?

Explanation: C. India has world's largest postal network with 1,55,837 post office (1,39,280 in rural areas). A post office serves an area of 21.09 sq kms and population of 6,602. Bharat Sanchar Nigam Limited (BSNL) was formed in the year 2000 and Telecom Regulatory Authority of India (TRAI) was established in the year 1997.

66. Which of the following pairs are correctly matched?
  1. Champadevi Sukla : Goldman Environmental Prize
  2. Dr. P. Sri Ramachanrudu : Vachaspati Puraskar.
  3. Ela Ramesh Bhatt : Lal Bahadur Shastri NationalAward for Excellence in Public Administration, Academic and Management.
  4. Upamanyu Chatterjee : Lalit Kala Ratna Award.

Select the correct answer using the code given below:

Explanation: B. Upamanyu Chatterjee is the winner of Shaitya Award winner and not Lalit Kala Award winner.

67. Which of the following pairs are correctly matched?
  1. A. Sportstars 1. The Hindu Publication Group
  2. B. Business-World 2. ABP Group
  3. C. The Week 3. Malayala Manorama Publication Group
  4. D. Reader's Digest 4. Indian Express Publication Group

Select the correct answer using the codes given below:

Explanation: B. Correct option is B.

68. Which one of the following was probed by the Liberhan Commission?

Explanation: D. The Liberhan Commission was commissioned by the Government of India to investigate the destruction of the disputed structure Babri Masjid in Ayodhya in 1992.

69. Whose autobiography is the book 'My Music, My Life'?

Explanation: A. 'My Music, My life' is the autobiography of Pandit Ravi Shankar.

70. Which one of the following airports in India is the first to be owned by a public limited company?

Explanation: B. Cochin International Airport is an international airport in the city of Kochi, in the state of Kerala. It is the first airport in India developed under a publicprivate partnership.

71. Which one of the following pairs is not correctly matched?

Explanation: B. Phalcon is radar system provided by Israel to India.

72. Who among the following is not recipient of the Bharat Ratna Award?

Explanation: D. Lata Mangeshkar 2001, Bismillah Khan 2001, and Satyajit Ray 1992.

73. Which one of the following pairs is not correctly matched?

Explanation: A. Slovenia's Capital is Ljublijana, and Bratislava is the capital of Slovakia.

74. What is the Universal Product Code (UPC) adopted for?

Explanation: C. The Universal Product Code (UPC) is a barcode symbology (i.e., a specific type of barcode) that is widely used in the United States, Canada, the United Kingdom, Australia, New Zealand, and in other countries for tracking trade items in stores.

75. What is the new administrative capital proposed for Myanmar?

Explanation: D. Pyinmana is a logging town and sugarcane refinery center in the Naypyidaw Union Territory of Myanmar. The administrative capital of Myanmar was officially moved to a militarized greenfield site two miles west of Pyinmana on November 6, 2005.

76. Consider the following statements:
  1. Kofi Annan, the UN Secretary General is from Nigeria.
  2. Kofi Annan is the First UN Secretary General to be appointed from the ranks of the United Nations Staff.
  3. Kofi Annan was appointed for a second term to the office of the UN Secretary General.

statements given above are correct?

Explanation: C. Kofi Annan of Ghana is the seventh Secretary- General of the United Nations. The first Secretary- General to be elected from the ranks of United Nations staff, he began his term on 1 January 1997. On 29 June 2001, acting on a recommendation by the Security Council, the General Assembly appointed him by acclamation to a second term of office, beginning on 1 January 2002 and ending on 31 December 2006.

77. Consider the following statements:
  1. The Nobel Prize Awarding Ceremony takes place on December 10, of every year.
  2. The Nobel Prize for Literature was added later on to the other five areas: Physics, Chemistry, Physiology or Medicine, Peace and Economics.

Which of the statement(s) given above is/are correct?

Explanation: A. Since 1901, the Nobel Prizes have been presented to the Laureates at ceremonies on 10 December, the anniversary of Alfred Nobel's death. As stipulated in the will of the Swedish-born inventor and international industrialist Alfred Nobel, which was opened after his death in 1896, the Nobel Prizes in Physics, Chemistry, Physiology or Medicine and Literature are awarded in Stockholm, Sweden, while the Nobel Peace Prize is awarded in Oslo, Norway.

78. In which one of the following films did the Nobel Laureate Pearl S. Buck collaborate?

Explanation: B. The screenplay of the film 'The Guide' had been written by Nobel Laureate Pearl S Buck.

79. Lectures from Colombo to Almora' is based on the experiences of which one of the following?

Explanation: D. Lectures from Colombo to Almora is a book of Swami Vivekananda based on his various lectures.

80. Who among the following wrote

Explanation: B. The Communist Manifesto was written by Karl Marx with the assistance of Friedrich Engels.

81. Who is Wole Soyinka?

Explanation: D. Akinwande Oluwole "Wole" Babatunde Soyinka is a Nigerian playwright and poet. He was awarded the 1986 Nobel Prize in Literature, the first African to be honored in that category.

82. Which one of the following pairs is not correctly matched?

Explanation: C. Genpact Limited is a multinational business process outsourcing and information technology services company, domiciled in Bermuda with executive headquarters in New York.

83. Match List-I with List-II and select the correct answer using the codes given below the lists:
  1. A. Harold Pinter 1. Director-General, FAG
  2. B. Jacques Diouf 2. President, European Commission
  3. C. Jose Manuel 3. World Bank appointed Barroso expert for adjudicating on the Baglihar Hydel Project
  4. D. Raymond Lafitte 4. Literature

Codes:

Explanation: C. Harold Pinter was a Nobel Prize-winning English playwright, screenwriter, director and actor. Jacques Diouf is a Senegalese diplomat who was Director- General of the United Nations' Food and Agriculture Organization from January 1994 to 31 December 2011. Jos

84. Which one of the following pairs is not matched?

Explanation: D. Eric Richard Kandel is an American neuropsychiatrist. He was a recipient of the 2000 Nobel Prize in Physiology or Medicine for his research on the physiological basis of memory storage in neurons.

85. Which one among the following was the first to legalize Euthanasia?

Explanation: C. In April 2002, the Netherlands became the first country to legalise euthanasia and assisted suicide. It imposed a strict set of conditions: the patient must be suffering unbearable pain, their illness must be incurable, and the demand must be made in "full consciousness" by the patient.

86. Which of the following pairs are correctly matched?
  1. Christopher Hovercraft Cockerell
  2. David Bushnell Submarine
  3. J. C. Perrier Steamship

Select the correct answer by using the codes given below:

Explanation: A. The Hovercraft invented by Christopher Cockerell is officially launched in Southampton. Sir Christopher Cockerel linvented the hovercraft, the vehicle which can travel across land or water on a cushion of air. David Bushnell is credited as the inventor of the submarine, which was first used to launch explosives against British ships during the American Revolution (1775-83).Steam ship was invented by J.C Perrier in the year 1775.

87. Where is Davos, the venue of the annual meeting of World Economic Forum; located?

Explanation: C. Davos is host to World Economic Forum, an annual meeting of global political and business elites and the home of huge ski resort is a town of Switzerland.

88. Consider the following statements:
  1. The Richter scale is a logarithmic scale and so an increase of 1 magnitude unit represents a factor of 10 times in amplitude.
  2. Each integer reading of the Richter scale has an energy 100 times that the previous integer reading.

Which of the statement(s) given above is/are correct?

Explanation: A. The magnitude of intensity of energy released by an earthquake is measured by Richter Scale. The number indicating magnitude on Richter Scale ranges between 0 and 9 but infact the scale has no upper limit of number because it is a logarithmic scale. But each integer reading of the Richeter Scale has an energy 32 times of previous integer reading. So the statement is not correct.

89. Which one of the following countries is not a member of the Nordic Council?

Explanation: D. The member of Nordic Council are Denmark, Finland, Iceland, Norway and Sweden with the autonomous territories Greenland, Faroe Island and Aland.

90. Which one of the following countries is not a member of the Commonwealth of Independent States (CIS)?

Explanation: C. Correct option is C.

91. Which one of the following countries is not a member of ASEAN?

Explanation: C. Correct option is C.

92. Match List-I (Person) with List-II (Organization/Area of Work) and select the correct answer using the codes given below the lists:
  1. A. Chandra Kochhar 1. Advertising
  2. B. Amrita Patel 2. Banking
  3. C. Indra Nooyi 3. Dairy Development
  4. D. Piyush Pandey 4. Pepsi Co

Codes:

Explanation: C. Correct option is C.

93. Which one among the following was awarded the CSIR.Diamond Jubilee Technology Award-2004 in September, 2005 by the Prime Minister of India?

Explanation: B. The "Midas Communications Technologies Private Limited" has been conferred with the CSIR Diamond Jubilee Technology Award 2004 for design, development and commercialization of cor DECT WLL (Wireless in Local Loop) access system and optima, a Fiber-in-Local Loop (FLL) access system. Midas Communication developed these products in collaboration with TeNeT (Telecommunication and Computer Network), lIT Madras (Chennai).

94. Who is the President of the Council of Scientific and Industrial Research?

Explanation: C. Prime Minister of India is the chairman of the Council of Scientific and Industrial Research. Council of Scientific and Industrial Research (CSIR) established in 1942, is an autonomous body and India's largest research and development (R&D) organisation. It operates as an autonomous body registered under the Registration of Societies Act of 1860.

95. Indian Airlines are have redesigned their logo which is a graphic wheel. This logo has been inspired from which one of the following?

Explanation: C. After the name change to Indian, the company's aircraft sported a new look inspired by the Sun Temple at Konark in Odisha.

96. Recently Uttar Pradesh and Madhya Pradesh governments signed a Memorandum of Understanding for the linking of two rivers as a link project. Which are these two rivers?

Explanation: B. Recently Uttar Pradesh and Madhya Pradesh Governments signed a memorandum of understanding for the linking of river Betwa and Ken. Both Ken and Betwa are the triburaties of Yamuna. According to the proposal 1020 million cubic metres of will be diverted from Ken Basin in a link canal that will be 231 km long. The canal will tranverse both states to provide irrigation and drinking water.

97. Who among the following was presented with the Sangeet Natak Academic Ratna Award, 2005 for her lifetime achievement in dance?

Explanation: D. Yamini Krishnamurthy.

98. What is IndoNext which was launched in January, 2005?

Explanation: D. BSE IndoNext is a joint initiative by BSE and Participating RSEs. In order to instill higher sense of involvement on the issues relating to management, business development, marketing and general operations of the BSE IndoNext platform, a "BSE IndoNext Council" has been constituted. This Council's duties have been defined in the Bye-laws of BSE and participating RSEs and operate under the overall supervision and control of the Governing Board of BSE.

99. Consider the following statements:
  1. In India, during the financial year 2004-2005 an increase of below 10% over the value of exports (in rupee terms) in the financial year 2003-2004 was reported.
  2. According to the WTO, India share in the world merchandise exports crosses 2% in the year 2005.

Which of the statement(s) given above is/are correct?

Explanation: D. Neither 1 nor 2.

100. With reference to the Government of India various programmes, what is Nirmal Gram Puraskar?

Explanation: D. Government of India (GOI) has been promoting sanitation coverage in a campaign mode to ensure better health and quality of life for people in rural India. To add vigour to its implementation, GOI launched an award based Incentive Scheme for fully sanitized and open defecation free Gram Panchayats, Blocks, Districts and States called "Nirmal Gram Puraskar" (NGP) in October 2003 and gave away the first awards in 2005 as a component of its flagship scheme Total Sanitation Campaign (TSC). Nirmal Gram Puraskar till 2011 was given by Ministry of Drinking Water and Sanitation (MoDWS), Government of India at all levels of PRIs that is Gram Panchayat, Block Panchayat and district Panchayat.

101. Consider the following statements:
  1. According to the census 2001, Kerala has the smallest gap in male and female literacy rates among the 28 states of India (Delhi and Pondicherry not included).
  2. According to the Census 2001, Rajasthan has literacy rate above the national average literacy rate.

Which of the statement(s) given above is/are correct?

Explanation: D. Mizoram is the state where male literacy is 90.7% and female literacy is 86.7%. So, Mizoram is the state of smallest gap in male and female literacy, where Kerala has 94.2% in male and 87.7% in female. In statement 2 literacy of Rajasthan is 60.4%, where national literacy is 64.8%.

102. Consider the following statements:
  1. The agreement on South Asian Free Trade Area (SAFTA) came into effect from 1st December, 2005.
  2. As per SAFT A agreement terms, India, Pakistan and Sri Lanka have to decrease their custom duties to the level of 0 to 5 per cent by the year 2013.

Which of the statement(s) given above is/are correct?

Explanation: D. Neither 1 nor 2.

103. Who did Croatia defeat in the Davis Cup-2005 Finals to win the Davis Cup-2005?

Explanation: B. The 2005 Davis Cup was the 94th edition of the most important tournament between nations in men's tennis. A total of 130 teams participated in the tournament. The final took place 2-4 December at the Sibamac Arena in Bratislava, Slovakia, with Croatia defeating Slovakia for their first title.

104. Which one of the following countries was not a participating team in the Football World Cup-2006?

Explanation: A. The 2006 FIFA World Cup was the 18th FIFA World Cup, the quadrennial international football world championship tournament. It was held from 9 June to 9 July 2006 in Germany, which won the right to host the event in July 2000. Russia did not participate in this event.

105. During the year 2000-01 which one of the following Industries recorded the highest growth rate in India?

Explanation: D. The steel industry recorded highest growth rate of 7%, electricity, generation growth rate was 4%, coal industry growth was 3.3%.

106. Who among the following have been the Union Finance Ministers of India?
  1. V P Singh
  2. R Venkataraman
  3. Y B Chavan
  4. Pranab Mukherjee

Select the correct answer using the code given below:

Explanation: D. VP Singh (1985-87), R Venkataraman (1980-82), YB Chavan (1971-75), Pranab Mukherjee (1982-85 ;Feb 2009-present).

107. Consider the following statements:
  1. North Atlantic Co-operation Council (NACC) is the name of the new organization which has replaced the North Atlantic Treaty Organization (NATO).
  2. The United States of America and the United Kingdom became the members of the NATO when it was formed in the year 1949.

Which of the statements given above is/are correct?

Explanation: B. Twelve countries were part of the founding of NATO: Belgium, Canada, Denmark, France, Iceland, Italy, Luxembourg, the Netherlands, Norway, Portugal, the United Kingdom and the United States. The North Atlantic Treaty Organization is also called the North Atlantic Alliance, is an intergovernmental military alliance based on the North Atlantic Treaty which was signed on 4 April 1949.

108. Match List-I with List-II and select the correct answer using the code given below the lists:
  1. A. Nancy Pelosi 1. WTO
  2. B. Margaret Chan 2. Speaker, US House of Representatives
  3. C. Pascal Lamy 3. WHO
  4. D. Steve Ballmer 4. Microsoft

Codes:

Explanation: C. A-2; B-3; C-1; D-4.

109. With reference to the international meetings held in the year 2006, which of the following pairs is/are correctly matched?
  1. NAM Summit : Havana
  2. APEC Meeting : Bangkok
  3. EU - India Summit : Helsinki
  4. UN Climate Change : Geneva Conference

Codes:

Explanation: B. 14th Nam Summit was held in Havana(Cuba) from 15-16 September 2006.The 17th EU India summit was held in Helsinkion 13 October 2006. 14th APEC Summit was held on 18-19 November 2006 in Hanoi(Vietnam). The 2006 United Nations Climate Change Conference took place between November 6 and 17, 2006 in Nairobi, Kenya.

110. Who among the following is Chile's first woman President?

Explanation: B. Michelle Bachelet was the first female president of Chile from 2006 to 2014.In December 2013, Bachelet was re-elected as President of Chile with over 62% of the vote. She is the first person since 1932 to win the presidency of Chile twice in competitive elections.

111. Match List I with List II and select the correct answer using the code given below the lists:
  1. A. Chevron 1. Wind energy
  2. B. AT & T 2. Oil
  3. C. AMD 3. Telephone, Internet
  4. D. Enercon GmbH 4. Microprocessor

Codes:

Explanation: C. Chevron Corporation is an American multinational energy corporationand one of the successor companies of Standard Oil headquartered in California. AT and T Inc. is an American multinational telecommunications corporation, headquartered at Whitacre Tower in downtown Dallas, Texas. Advanced Micro Devices, Inc. is an American worldwide semiconductor company based in Sunnyvale, California, United States, that develops computer processors. Enercon GmbH is the fourth-largest wind turbine manufacturer in the world based in Germany.

112. Which one of the following pairs is not correctly matched?

Explanation: C. Nick was appointed as a lecturer in Bioengineering at Southampton University in January 2011.

113. Yom Kippur War was fought between which sides countries?

Explanation: C. The Yom Kippur War, also known as the 1973 Arab-Israeli War, was a war fought by the coalition of Arab states led by Egypt and Syria against Israel from October 6 to 25, 1973.

114. Who among the following is considered as the inventor of the World Wide Web (WWW)?

Explanation: C. "Tim" Berners-Lee is best known as the inventor of the World Wide Web.

115. Where are the headquarters of the Organization of the Islamic Conference (OIC) located?

Explanation: B. The Organization of Islamic Cooperation (OIC) is the second largest inter-governmental organization after the United Nations. Its headquarters is situated in Jeddah, Saudi Arabia.

116. How is Gabriel Garcia Marquez well-known as

Explanation: C. Gabriel Garcia Marquez was a Colombian novelist, short-story writer, screenwriter and journalist. He was awarded with Nobel Prize in Literature in 1982.

117. Where was the first conference of the Pugwash Conferences on Science and World Affairs held in the year 1957?

Explanation: C. The Pugwash Conferences on Science and World Affairs is an international organization that brings together scholars and public figures to work toward reducing the danger of armed conflict and to seek solutions to global security threats. It was founded in 1957 by Joseph Rotblat and Bertrand Russell in Pugwash, Nova Scotia, Canada.

118. Consider the following statements:
  1. The series of the International Paper Sizes is based on A0 size whose area is 0.5 m2 (approximately).
  2. The area of A4 size paper is 1/8th of that of the A0 size paper.

Which of the statements given above is/are correct?

Explanation: D. Correct option is D.

119. Which one of the following pairs is not correctly matched?

Explanation: C. A.M. Khusro was Chairman of eleventh Finance Commission of India.

120. Which one of the following is the correct chronological order of the formation of the following as full States of the Indian Union?

Explanation: B. Nagaland-1962, Haryana-1966, Sikkim-1975, Arunachal Pradesh-1986.

121. In which one of the following cities is the Global Automotive Research Centre being set up?

Explanation: A. The Global Automotive Research Center (GARC), Chennai, is situated in the SIPCOT Industrial Growth Center at Orgadam near Chennai in the close proximity of manufacturing facility of Indian and Global automotive giants.

122. Who among the following served as the Chief Economist of the International Monetary Fund?

Explanation: D. Raghuram Govind Rajan is currently the Eric J. Gleacher Distinguished Service Professor of Finance at the Booth School of Business at the University of Chicago. He is also an honorary economic adviser to Prime Minister of India Manmohan Singh (appointed 2008). He previously was the chief economist of the International Monetary Fund and headed a committee appointed by the Planning Commission on financial reforms in India.

123. Match List-I with List-II and select the correct answer using the code given below the lists:
  1. A. Bhajan Sopori l. Bharatnatyam dancer
  2. B. Birju Maharaj 2. Exponent of Santoor
  3. C. Priyadarsini 3. Mridangam maestro
  4. D. TV Gopalakrishnan 4. Kathak dancer

Codes:

Explanation: C. A-2; B-4; C-1; D-3.

124. Who among the following wrote the book

Explanation: B. P V Narasimha Rao.

125. The Pulitzer Prize is associated with which one of the following?

Explanation: C. The Pulitzer Prize is an award for achievements in newspaper and online journalism, literature, and musical composition in the United States. It was established in 1917 by provisions in the will of American (Hungarian-born) publisher Joseph Pulitzer and is administered by Columbia University in New York City.

126. Cristina Kirchner succeeded her husband to become President of a South American country. Which is that country?

Explanation: B. Argentina.

127. Consider the following statements:
  1. Orange Prize is awarded to a work of published fiction in English by a woman.
  2. Pulitzer Prize is awarded by the Commonwealth Foundation to a citizen of any Commonwealth country for any literary work in English.

Which of the statements given above is/are correct?

Explanation: A. only 1.

128. In the year 2007, an earthquake led to massive radioactive water leakage in the largest nuclear plant in the world. In which country did it occur?

Explanation: C. Japan.

129. Where was the World Summit on Sustainable Development (Rio + 10) held?

Explanation: C. Johannesburg.

130. Where was the first desalination plant in India to produce one lakh litres freshwater per day based on low temperature thermal desalination principle commissioned?

Explanation: A. Kavaratti.

131. Hand-in-Hand 2007', a joint anti terrorism military training was held by the officers of the Indian Army and officers of Army of which one of the following countries?

Explanation: A. China.

132. Kim Dae-jung won the Nobel for Peace. He is from which one of the following countries?

Explanation: C. South Korea.

133. How is the United Nations Monetary and Financial Conference where in the agreements were signed to set up IBRD, GATT and IMF, commonly known?

Explanation: B. Bretton woods Conference.

134. Match List-I with List-II and select the correct answer using the code given below the Lists:
  1. A. Nagender Singh 1. Chief Election Commissioner of India
  2. B. A N Ray 2. President, International Court of Justice
  3. C. R. K Trivedi 3. Chief Justice of India
  4. D. Ashok Desai 4. Attorney General of India

Codes:

Explanation: B. Ashok Desai was Attorney General of India (1996-98); Nagendra Singh was President of International Court of Justice (1985-88); AN Ray was CJI (1973-77); RK Trivedi was Chief Election Commissioner (1982-85).

135. In August 2006, the Government of India notified the Rural Electrification Policy. This policy aims at provision of access to all households by which year?

Explanation: B. Its Goals include provision of access to electricity to all households by the year 2009, quality and reliable power supply at reasonable rates and minimum lifeline consumption of 1 unit/household/day as a merit good by year 2012.

136. Recently, the manuscripts of which one of the following have been included in the UNESCO's Memory of World Register?

Explanation: D. The Rig Veda manuscripts have been selected for inscription in UNESCO's "Memory of the World" Register 2007. The program for the 'Memory of the World' was started by the UNESCO to honor significant landmarks in the documentary heritage and record them in its 'Memory of the World Register' as world's inheritance.

137. Match List-I with List-II and select the correct answer using the code given below the Lists:
  1. A. K P Singh 1. Banking
  2. B. Vikram Pandit 2. Fiction writing
  3. C. Roopa Farooki 3. Retail business
  4. D. Kishore Biyani 4. Real estate

Codes:

Explanation: D. A-4; B-1; C-2; D-3.

138. For which one of the following reforms was a Commission set up under the Chairmanship of Veerappa Moily by the Government of India?

Explanation: D. Administrative Reforms.

139. From which one of the following did India buy the Barak anti-missile defence systems?

Explanation: A. Barak is an Israeli surface-to-air missile (SAM) designed to be used as a ship-borne point-defense missile system against aircraft, anti-ship missiles, and UAVs. India and Israel jointly developed and produced it.

140. In the context of the Indian defence, what is 'Dhruv'?

Explanation: C. Hindustan Aeronautics Limited (HAL) has developed the Dhruv advanced light helicopter (ALH), a light (5.5t class) multirole and multi-mission helicopter for army, air force, navy, coastguard and civil operations for both utility and attack roles by day and night.

141. Which one of the following Union Ministries is implementing the Biodiese1 Mission (as Nodal Ministry)?

Explanation: D. The Committee on Development of Bio-fuels setup by the Planning Commission in July, 2002, in its report of April, 2003, recommended launching of the National Commission on Bio-diesel. Ministry of Rural development was designated as the Nodal Ministry for this Mission.

142. Match List-I with List-II and select the correct answer using the code given below the Lists:
  1. A. Sabyasachi Mukherjee 1. Microfinance loans
  2. B. Aniruddha Bahal 2. Pharmaceuticals
  3. C. Vikram Akula 3. Fashion designing
  4. D. Yusuf Hamied 4. Investigative journalism

Codes:

Explanation: B. Sabyasachi Mukherjee is an Indian fashion designer from Kolkata. Aniruddha Bahal is an Indian journalist, author, founder and editor of the online magazine Cobrapost.com. Vikram Akula is a pioneer in market-based approaches to financial inclusion. Yusuf Khwaja Hamied is an Indian scientist and chairman of Cipla.

143. Ogaden region has been a source of conflict between which countries?

Explanation: D. Ogaden region has been a source of conflict between Ethiopia and Somalia.

144. Match List-I with List-II and select the correct answer using the code given below the Lists:
  1. A. Foreign Intelligence Service 1. Israel
  2. B. Ministry of State Security 2. Britain
  3. C. Secret Intelligence Service 3. China
  4. D. The Mossad 4. Russia

Codes:

Explanation: C. The Foreign Intelligence Service is Russia's external intelligence agency, mainly for civilian affairs. The Ministry of State Security (MSS) is the intelligence agency and security agency of the People's Republic of China. The Secret Intelligence Service(commonly known as MI6) is the British intelligence agency which supplies the British Government with foreign intelligence. Mossad is the national intelligence agency of Israel.

145. Norman Ernest Borlaug who is regarded as the father of the Green Revolution in India is from which country?

Explanation: A. Norman Ernest Borlaug was an American biologist who has been called "the father of the Green Revolution.

146. Match List-I with List-II and select the correct answer using the code given below the Lists:
  1. A. Amnesty International 1. Viskuli
  2. B. Commonwealth of Independent States(CIS) 2. Paris
  3. C. Danube Commission 3. Peter Benenson
  4. D. Europe Space Agency 4. Belgrade Convention

Codes:

Explanation: A. A-3; B-1; C-4; D-2.

147. How much is one barrel of oil approximately equal to

Explanation: B. The one barrel of crude consists of approximately 159 Liters.

148. Which of the following pairs is/are correctly matched?
  1. Francis Collins : Mapping human genome
  2. Sergey Brin : Google Search Engine
  3. Jimmy Wales : Wikipedia

Select the correct answer using the code given below:

Explanation: D. All are correctly matched.

149. Who among the following is the founder of World Economic Forum?

Explanation: A. The foundation was founded in 1971 by Klaus Schwab, a German-born business professor at the University of Geneva. Originally named the European Management Forum, it changed its name to the World Economic Forum in 1987.

150. Consider the following pairs:
  1. Cannes : France
  2. Davos : Denmark
  3. Roland Garros : The Netherlands

Which of the pairs given above is/are correctly matched?

Explanation: A. Davos is a town in Switzerland and Roland Garros is in England.

151. Who of the following is the author of the book "The Audacity of Hope"?

Explanation: B. The Audacity of Hope is a 2006 autobiographical work by Barack Obama.

152. A present group of nations known as G-8 started first as G-7. Which one among the following was not one of them?

Explanation: D. The Group of Seven (G7) is an informal bloc of industrialized democracies

153. Consider the following countries:
  1. Switzerland
  2. Malta
  3. Bulgaria

Which of the above are members of European Union?

Explanation: B. The EU countries are: Austria, Belgium, Bulgaria, Croatia, Republic of Cyprus, Czech Republic, Denmark, Estonia, Finland, France, Germany, Greece, Hungary, Ireland, Italy, Latvia, Lithuania, Luxembourg, Malta, Netherlands, Poland, Portugal, Romania, Slovakia, Slovenia, Spain, Sweden and the UK.

154. Consider the following pairs:
  1. Asian Development Bank : Tokyo
  2. Asia-Pacific Economic Cooperation : Singapore
  3. Association of South East Asian Nations : Bangkok

Which of the above pairs is/are correctly matched?

Explanation: B. The headquarters of Asia pacific economic cooperation is in Singapore. The headquarters of Association of Southeast Asia Nations (ASEAN) is in Jakarta, Indonesia. The headquarters of Asian Development Bank is in the city of Philippines.

155. With reference to the United Nations, consider the following statements:
  1. The Economic and Social Council (ECOSOC) of UN consists of 24 member States.
  2. It is elected by a 2/3rd majority of the General Assembly for a 3-year term.

Which of the statements given above is/are correct?

Explanation: B. The United Nations Economic and Social Council has 54 member states which are elected by the United Nations General Assembly for overlapping three-year terms. They are elected by the General Assembly for overlapping three-year terms. Seats on the Council are allotted based on geographical representation with fourteen allocated to African States, eleven to Asian States, six to Eastern European States, ten to Latin American and Caribbean States as well as thirteen to Western European and other States. They are elected by 2/3rd majority of the General Assembly.

156. Which one of the following pairs of countries fought wars over a region called Ogaden?

Explanation: B. Ethiopia and Somalia fought wars for Ogaden.

157. Consider the following countries:
  1. Brunei Darussalam
  2. East Timor
  3. Laos

Which of the above is/are member/members of ASEAN?

Explanation: C. The Association of Southeast Asian Nations, is a geo-political and economic organization of 10 countries located in Southeast Asia, which was formed on 8 August 1967 by Indonesia, Malaysia, the Philippines, Singapore and Thailand. Since then, membership has expanded to include Brunei, Burma (Myanmar), Cambodia, Laos, and Vietnam.

158. Other than Venezuela, which one among the following from South America is a member of OPEC?

Explanation: C. The Organization of the Petroleum Exporting Countries (OPEC) is a cartel of twelve developing countries made up of Algeria, Angola, Ecuador, Iran, Iraq, Kuwait, Libya, Nigeria, Qatar, Saudi Arabia, the United Arab Emirates, and Venezuela.

159. Consider the following pairs:
  1. ABN Amro Bank : USA
  2. Barclays Bank : UK
  3. Kookmin Bank : Japan

Which of the above pairs is/are correctly matched?

Explanation: B. ABN AMRO Bank N.V. is a Dutch bank with headquarters in Amsterdam, the Netherlands. Barclays PLC is a global financial services company headquartered in London, United Kingdom. Kookmin Bank is the largest bank by both asset value and market capitalization in South Korea.

160. Consider the following pairs:
  1. BMW AG : USA
  2. Daimler AG : Sweden
  3. Renault S.A. : France
  4. Volkswagen AG : Germany

Which of the pairs given above is/are correctly matched?

Explanation: B. BMW AG and Daimler AG are Head quartered in Germany.

161. The Security Council of UN consists of 5 permanent members, and the remaining 10 members are elected by the General Assembly for a term of

Explanation: B. Five permanent members: China, France, Russian Federation, the United Kingdom, and the United States, and ten non-permanent members elected for two-year terms by the General Assembly (with end of term date).

162. The brothers Umakant and Ramakant Gundecha are

Explanation: A. Umakant Gundecha and Ramakant Gundecha, known as the Gundecha Brothers, are leading Dagarvani dhrupad singers.

163. Which one of the following is not a drug/ pharma company?

Explanation: A. Chevron Corporation is an American multinational energy corporation.

164. Match List I with List II and select the correct answer using the code given below the lists:
  1. A. Amrita Sher-Gil 1. Dancer
  2. B. Bhimsen Joshi 2. Painter
  3. C. Rukmini Devi Arundale 3. Poet
  4. D. Suryakant Tripathi Nirala 4. Singer

Codes:

Explanation: B. A-2; B-4; C-1; D-3.

165. Match List-I with List-II and select the correct answer using the code given below the lists:
  1. A. In Custody 1. Amartya Sen
  2. B. Sea of Poppies 2. Amitav Ghosh
  3. C. The Argumentative Indian 3. Anita Desai
  4. D. Unaccustomed Earth 4. Jhumpa Lahiri

Codes:

Explanation: C. A-3; B-2; C-1; D-4

166. Who of the following is the author of a collection of poems called "Golden Threshold"?

Explanation: C. Golden Thresold is a collection of poems by Sarojini Naidu.

167. Consider the following pairs:
  1. Ashok Leyland : Hinduja Group
  2. Hindalco Industries : A.V. Birla Group
  3. Suzlon Energy : Punj Lloyd Group

Which of the pairs given above is/are correctly matched?

Explanation: A. The Hinduja Group is a global conglomerate company headquartered in London, United Kingdom. Ashok Leyland is one of its companies. Hindalco Industries Limited, the metals flagship company of the Aditya Birla Group, is the world's largest aluminium rolling company and one of the biggest producers of primary aluminium in Asia.

168. Under the administration of which one of the following is the Department of Atomic Energy?

Explanation: A. The DAE is under the direct control of PM with its Headquarter in Mumbai.)

169. Where is the headquarters of Animal Welfare Boards of India located?

Explanation: B. The Animal Welfare Board of India was set up in 1962 with its headquarter at Madras, under the provisions of the "Prevention of Cruelty to Animals Act 1960" (PCA Act. 1960).

170. Consider the following statements:
  1. The National School of Drama was set up by Sangeet Natak Akademi in 1959.
  2. The highest honour conferred by the Sahitya Akademi on a writer is by electing him its Fellow.

Which of the statements given above is/are correct?

Explanation: C. Both 1 and 2

171. For outstanding contribution to which one of the following fields is Shanti Swarup Bhatnagar Prize given

Explanation: C. The Shanti Swarup Bhatnagar Prize for Science and Technology (SSB) is a science award in India given annually by the Council of Scientific and Industrial Research (CSIR) for notable and outstanding research, applied or fundamental, in biology, chemistry, environmental science, engineering, mathematics, medicine and Physics.

172. Among the following Presidents of India, who was also the Secretary General of Non-Aligned Movement for some period?

Explanation: C. Giani Zail Singh was the 7th president of India. He also became 9th Secretary General of Non Aligned Movement.

173. Match List-I with List-II and select the correct answer using the code given below the lists:
  1. A. MandakiniAmte 1. Theatre direction
  2. B. Neelam Mansingh 2. Social service and Chowdhary community leadership
  3. C. Romila Thapar 3. Dance
  4. D. Vanashree Rao 4. History writing

Codes:

Explanation: A. A-2; B-1; C-4; D-3

174. With reference to Stree Shakti Puraskar, for which one of the following is "Devi Ahilya Bai Holkar Award" given

Explanation: A. Administrative Skills.

175. In which State is the Buddhist site Tabo Monastery located?

Explanation: B. Tabo Monastery is located in the Tago village of Spiti Valley, Himachal Pradesh.

176. Among the following, who are the Agaria community?

Explanation: D. The Agaria are a Muslim community found in the state of Gujarat in India.

177. Which one of the following pairs is not correctly matched?

Explanation: B. Pamela Jelimo is a Kenyan middle-distance runner, specialising in the 800 metres.

178. Who of the following is a football player?

Explanation: D. Nicolas Sebastien Anelka is a French footballer.

179. Among the following which one is not a football club?

Explanation: D. Monte Carlo officially refers to an administrative area of the Principality of Monaco specifically the ward of Monte, where the Monte Carlo Casino is located.

180. Consider the following pairs:
  1. Anand Pawar : Chess
  2. Akhil Kumar : Boxing
  3. Shiv Shankar Prasad Chowrasia : Golf

Which of the pairs given above is/are correctly matched?

Explanation: B. Anand Pawar is a male Indian badminton player who competed at 2012 Japan Super Series.

181. A very big refugee camp called Dadaab, recently in the news, is located in

Explanation: B. Kenya.

182. Consider the following names:
  1. Ike
  2. Kate
  3. Gustav

Which of the above are the names of hurricanes that had occurred very recently?

Explanation: C. Option C is correct.

183. In the latter half of the year 2008, which one of the following countries pulled out of Commonwealth of Independent States?

Explanation: C. Georgia.

184. Match List-I (Ethnic Community) with List-II (Country) and select the correct answer using the codes given below the lists:
  1. A. Apatani 1. China
  2. B. Dyak 2. India
  3. C. Dinka 3. Indonesia
  4. D. Uighur 4. Sudan

Codes:

Explanation: A. A-2; B-3; C-4; D-1.

185. Match List-I (Organisation) with List-II (Headquarters) and select the correct answer using the codes below the lists:
  1. A. International Atomic Energy Agency 1. Brussels
  2. B. International Telecommunication 2. Geneva
  3. C. Union Council of the European Union 3. Paris
  4. D. OECD 4. Vienna

Codes:

Explanation: D. The headquarter of International atomic Energy Agency is in Viena. The headquarters of International Telecommunication Union is in Geneva, Headquarters of Council of the European Union is in Brussels and Headquarters of Organisation for Economic Cooperation and Development is in Paris.

186. "World Development Report" is an annual publication of:

Explanation: B. WDR is an annual publication of World bank i.e. International Bank of Reconstruction and Development

187. Consider the following statements:
  1. The World Intellectual Property Organisation (WIPO) is a specialized agency of United Nations System of Organisations
  2. WIPO has its headquarters at Rome
  3. The Trade Related Aspects of Intellectual Property Rights (TRIPS) Agreement is binding on all WTO members
  4. Least developed country members of WTO are not required to apply the provisions of TRIPS Agreement for a period of 20 years from the general date of application of the Agreement

Which of these statements are correct?

Explanation: D. WIPO has its HQ at Geneva.

188. The world's highest ground based telescopic observatory is located in:

Explanation: B. The world's highest ground based telescopic observatory is located at Leh in India.

189. Chapchar Kut is a festival celebrated in the state of:

Explanation: C. Chapchar kut is a festival celebrated in the state of Mizoram.

190. In the context of global oil prices, "Brent crude oil" is frequently referred to in the news. What does this term imply?
  1. It is a major classification of crude oil.
  2. It is sourced from North sea.
  3. It does not contain sulphur.

Which of the statements given above is/are correct?

Explanation: B. It contains sulphur which is even higher than WTI (West Texas Intermediate) crude oil.

191. The "New START" treaty was in the news. What is this treaty?

Explanation: A. It is a bilateral strategic nuclear arms reduction treaty between the USA and the Russian Federation.

192. South-East Asia has captivated the attention of global community over space and time as a geo strategically significant region. Which among the following is the most convincing explanation for this global perspective?

Explanation: D. Its location between the Pacific and Indian oceans and its pre-eminent maritime character.

193. Recently, the USA decided to support India
  1. The Australia group is an informal arrangement which aims to allow exporting countries to minimize the risk of assisting chemical and biological weapons proliferation, whereas the Wassenaar arrangement is a formal group under the OECD holding identical objectives.
  2. The Australia group comprises predominantly of Asian, African and north American countries, whereas the member countries of Wassenaar arrangement are predominantly from the European union and American continents.

Which of the statements given above is/are correct?

Explanation: D. The Australia Group (AG) is an informal association that works on the basis of consensus. It aims to allow exporters or transshipment countries to minimize the risk of further proliferation of chemical and biological weapons (CBW). There are not many Asian/African countries in the Australian Group. There are 43 members including India in this group.

194. Consider the following statements:
  1. North Atlantic Co-operation Council (NACC) is the name of the new organization which has replaced the North Atlantic Treaty Organization (NATO).
  2. The United States of America and the United Kingdom became the members of the NATO when it was formed in the year 1949.

Which of the statements given above is/are correct?

Explanation: B. Twelve countries were part of the founding of NATO: Belgium, Canada, Denmark, France, Iceland, Italy, Luxembourg, the Netherlands, Norway, Portugal, the United Kingdom and the United States. The North Atlantic Treaty Organization is also called the North Atlantic Alliance, is an intergovernmental military alliance based on the North Atlantic Treaty which was signed on 4 April 1949.

195. With reference to the international meetings held in the year 2006, which of the following pairs is/are correctly matched?
  1. NAM Summit : Havana
  2. APEC Meeting : Bangkok
  3. EU - India Summit : Helsinki
  4. UN Climate Change : Geneva Conference

Codes:

Explanation: B. 14th Nam Summit was held in Havana(Cuba) from 15-16 September 2006.The 17th EU India summit was held in Helsinkion 13 October 2006. 14th APEC Summit was held on 18-19 November 2006 in Hanoi(Vietnam). The 2006 United Nations Climate Change Conference took place between November 6 and 17, 2006 in Nairobi, Kenya.

196. Who among the following is Chile's first woman President?

Explanation: B. Michelle Bachelet was the first female president of Chile from 2006 to 2014.In December 2013, Bachelet was re-elected as President of Chile with over 62% of the vote. She is the first person since 1932 to win the presidency of Chile twice in competitive elections.

197. Which one of the following pairs is not correctly matched?

Explanation: C. Nick was appointed as a lecturer in Bioengineering at Southampton University in January 2011.

198. Yom Kippur War was fought between which sides countries?

Explanation: C. The Yom Kippur War, also known as the 1973 Arab-Israeli War, was a war fought by the coalition of Arab states led by Egypt and Syria against Israel from October 6 to 25, 1973.

199. Who among the following is considered as the inventor of the World Wide Web (WWW)?

Explanation: C. "Tim" Berners-Lee is best known as the inventor of the World Wide Web.

200. Consider the following:
  1. Hotels and restaurants
  2. Motor transport undertakings
  3. Newspaper establishments
  4. Private medical institutions

The employees of which of the above can have the Social Security coverage under Employees State Insurance Scheme?

Explanation: D. D is correct option.

201. Recently, there has been a concern over the short supply of a group of elements called 'rare earth metals'. Why?
  1. China, which is the largest producer of these elements, has imposed some restrictions on their export.
  2. Other than China, Australia, Canada and Chile, these elements are not found in any country.
  3. Rare earth metals are essential for the manufacture of various kinds of electronic items and there is a growing demand for these elements.

Which of the statements given above is/are correct?

Explanation: B. It is based on renewable resources like sugar, molasses, rice, corn and potato starches.

202. Graphene is frequently in news recently. What is its importance?
  1. It is a two-dimensional material and has good electrical conductivity.
  2. It is one of the thinnest but strongest materials tested so far.
  3. It is entirely made of silicon and has high optical transparency.
  4. It can be used as 'conducting electrodes' required for touch screens, LCDs and organic LEDs.

Which of the statements given above are correct?

Explanation: C. Graphene is a two-dimensional material and has good electrical conduction. It is one of the thinnest but strongest materials tested so far. It can be used for conducting electrodes required for touch screens, LCDs and organic LEDs.

203. With reference to National Rural Health Mission, which of the following are the jobs of 'ASHA', a trained community health worker?
  1. Accompanying women to the health facility for antenatal care checkup
  2. Using pregnancy test kits for early detection of pregnancy
  3. Providing information on nutrition and immunization
  4. Conducting the delivery of baby

Select the correct answer using the codes given below:

Explanation: A. Conducting the delivery of baby is not the work of ASHA.

204. How does the National Rural Livelihood Mission seek to improve livelihood options of rural poor?
  1. By setting up a large number of new manufacturing industries and agribusiness centres in rural areas
  2. By strengthening 'self-help groups' and providing skill development
  3. By supplying seeds, fertilizers, diesel pump-sets and micro-irrigation equipment free of cost to farmers

Select the correct answer using the codes given below :

Explanation: B. B is correct option.

205. The endeavour of 'Janani Suraksha Yojana' Programme is
  1. to promote institutional deliveries
  2. to provide monetary assistance to the mother to meet the cost of delivery
  3. to provide for wage loss due to pregnancy and confinement

Which of the statements given above is/are correct?

Explanation: A. The endeavour of Janani Suraksha Yojana programme are: (i) to promote institutional deliveries (ii) to provide monetary assistance to the mother to meet the cost of delivery.

206. To meet its rapidly growing energy demand, some opine that India should pursue research and development on thorium as the future fuel of nuclear energy. In this context, what advantage does thorium hold over uranium?
  1. Thorium is far more abundant in nature than uranium.
  2. On the basis of per unit mass of mined mineral, thorium can generate more energy compared to natural uranium.
  3. Thorium produces less harmful waste compared to uranium.

Which of the statements given above is/are correct?

Explanation: D. Thorium fuel generates no new bomb-usable material in the waste profile; the waste consists of the Radio isotope Uranium-233, or U233, which is virtually impossible to weaponize/ Thorium fuel will generate more energy per unit of mass than uranium fuel by a factor of approximately 30. Thorium is four times more abundant in nature than uranium, and is widely distributed throughout the Earth's crust. All statements in this question are correct.

207. With reference to 'stem cells', frequently in the news, which of the following statements is/are correct?
  1. Stem cells can be derived from mammals only.
  2. Stem cells can be used for screening new drugs.
  3. Stem cells can be used for medical therapies

Select the correct answer using the codes given below :

Explanation: B. Stem cells can be used for medical therapies also can be used for screening new drugs.

208. What are the reasons for the people's resistance to the introduction of Bt brinjal in India?
  1. Bt brinjal has been created by inserting a gene from a soil fungus into its genome.
  2. The seeds of Bt brinjal are terminator seeds and therefore, the farmers have to buy the seeds before every season from the seed companies.
  3. There is an apprehension that the consumption of Bt brinjal may have adverse impact on health.
  4. There is some concern that the introduction of Bt brinjal may have adverse effect on the biodiversity.

Select the correct answer using the codes given below :

Explanation: B. 2nd and 3rd statements are correct hence the option (B) is right.

209. Consider the following pairs:
  1. Limboo (Limbu) : Sikkim
  2. Karbi : Himachal Pradesh
  3. Dongaria Kondh : Odisha
  4. Bonda : Tamil Nadu

Which of the above pairs are correctly matched?

Explanation: A. Limbu tribe is from Sikkim and Dongaria Kondh tribe is from Odisha ( Karbi is from Assam and Bonda is in Odisha).

210. The efforts to detect the existence of Higgs boson particle have become frequent news in the recent past. What is/are the importance / importances of discovering this particle?
  1. It will enable us to understand as to why elementary particles have mass.
  2. It will enable us in the near future to develop the technology of transferring matter from one point to another without traversing the physical space between them.
  3. It will enable us to create better fuels for nuclear fission.

Select the correct answer using the codes given below.

Explanation: A. The discovery of Higgs boson is important because it explains why sub atomic particles have mass. According to the scientists, Higgs boson is the only particle which explains how the basic building blocks of matter interact.

211. Consider the following countries:
  1. Denmark
  2. Japan
  3. Russian Federation
  4. United States of America

Which of the above are the members of the 'Arctic Council'?

Explanation: D. The Arctic Council is a high-level intergovernmental forum that addresses issues faced by the Arctic governments and the indigenous people of the Arctic. It has eight member countries: Canada, Denmark, Finland, Iceland, Norway, Russia, Sweden, and the United States.

212. Chaitra 1 of the national calendar based on the Saka Era corresponds to which one of the following dates of the Gregorian calendar in a normal year of 365 days?

Explanation: A. Chaitra 1 of the national calendar based on the Saka Era corresponds to 22nd March of the Gregorian calendar in a normal year of 355 days and on 21 March in leap year.

213. Which of the following organizations brings out the publication known as 'World Economic Outlook'?

Explanation: A. The World Economic Outlook (WEO) database contains selected macroeconomic data series from the statistical appendix of the World Economic Outlook report prepared by IMF.

214. With reference to the cultural history of India, the term 'Panchayatan' refers to

Explanation: C. Panchayatana is a style of temple construction that has a central shrine surrounded by four subsidiary shrines.

215. With reference to a grouping of countries known as BRICS, consider the following statements:
  1. The First Summit of BRICS was held in Rio de Janeiro in 2009.
  2. South Africa was the last to join the BRICS grouping.

Which of the statements given above is/are correct?

Explanation: B. BRICS is the acronym for an association of five major emerging national economies: Brazil, Russia, India, China, and South Africa. The grouping was originally known as "BRIC" before the inclusion of South Africa in 2010. The BRIC first formal summit held in Yekaterinburg, commenced on 16 June, 2009.

216. Recently, a series of uprisings of people referred to as 'Arab Spring' originally started from

Explanation: D. The Arab Spring is a revolutionary wave of demonstrations, protests and civil wars in the Arab world that began on 18 December 2010 and spread throughout the countries of the Arab League. It originally started from Tunisia.

217. Consider the following countries
  1. China
  2. France
  3. India
  4. Israel

Which among the above are Nuclear Weapons States as recognized by the Treaty on the Non Proliferation of Nuclear Weapons, commonly known as Nuclear Non-Proliferation Treaty (NPT)?

Explanation: A. NPT designated nuclear weapon states are China, France, Russia, United Kingdom and the United States. The NPT is a landmark international treaty whose objective is to prevent the spread of nuclear weapons and weapons technology and to promote cooperation in the peaceful uses of nuclear energy and to further the goal of achieving nuclear disarmament and complete disarmament. It is opened for signature in 1968 and the Treaty entered into force in 1970.

218. Amnesty International is

Explanation: B. Amnesty International is a non-governmental organisation focused on human rights. It was founded in London in 1961 by the lawyer Peter Benenson. It draws attention to human rights abuses and campaigns for compliance with international laws and standards.

219. Which one of the following issues the 'Global Economic Prospects' report periodically?

Explanation: D. The World Bank issues the 'Global Economic Prospects' report periodically.

220. India is a member of which among the following?
  1. Asia-Pacific Economic Cooperation
  2. Association of South-East Asian Nations
  3. East Asia Summit

Select the correct answer using the code given below.

Explanation: B. At present, India is not a member of APEC and ASEAN. The East Asia Summit (EAS) is a forum held annually by leaders of 16 countries in the East Asian region. The first summit was held in Kuala Lumpur (Malaysia) on 14 December 2005. India is the member of this forum.

221. Which of the following statements is/are correct regarding National Innovation Foundation-India (NIF)?
  1. NIF is an autonomous body of the Department of Science and Technology under the Central Government.
  2. NIF is an initiative to strengthen the highly advanced scientific research in India's premier scientific institutions in collaboration with highly advanced foreign scientific institutions.

Select the correct answer using the code given below.

Explanation: A. National Innovation Foundation India (NIF) is an autonomous body under the Department of Science and Technology , Government of India. It was set up in February 2000 at Ahmadabad, Gujarat, India to provide institutional support for scouting, spawning, sustaining and scaling up the grassroots innovations across the country.

222. Consider the following pairs:
  1. Srisailam : Nallamala Hills
  2. Omkareshwar : Satmala Hills
  3. Pushkar : Mahadeo Hills

Which of the above pairs is/are correctly matched?

Explanation: A. The town of Srisailam is reputed for the shrine of Lord Mallikarjuna on the flat top of Nallamala Hills. Omkareshwar is one of the Dwadasa jyotirlingas situated on the Mandhata hills in Vindhya Mountains of Madhya Pradesh. Pushkar is located along Ratnagiri Hills in Rajasthan.

223. The problem of international liquidity is related to the non-availability of

Explanation: C. Dollars and hard currencies are used for payments in international trade, investment and liquidation of international debt. So if there is shortage of liquidity, this means scarcity of Dollars and hard currencies to carry on the above transactions. That is why even SDR was introduced by IMF.

224. The 'Fortaleza Declaration', recently in the news, is related to the affairs of

Explanation: B. Fortaleza declaration was part of 6th BRICS summit which resulted in the official inauguration of the New Development Bank.

225. Beijing Declaration and Platform for Action often seen in the news, is

Explanation: C. The Beijing Declaration and Platform for Action was adopted by the Fourth World Conference on Women in 1995. The Platform for Action reaffirms the fundamental principle that the rights of women and girls are an "inalienable, integral and indivisible part of universal human rights." The Platform for Action also calls upon governments to take action to address several critical areas of concern, among them violence against women.

226. Among the following, which were frequently mentioned in the news for the outbreak of Ebola virus recently?

Explanation: B. Guinea, Sierra Leone and Liberia were in news for the outbreak of Ebola virus recently.

227. The area known as 'Golan Heights' sometimes appears in the news in the context of the events related to

Explanation: B. Golan Heights are located in Syria. However, the two-thirds of the western region are currently occupied by Israel, whereas the one-third of eastern region is controlled by Syria.

228. The term 'Goldilocks Zone' is often seen in the news in the context of

Explanation: C. Description is righ. "Goldilocks Zone," is the region around a star that has just the right conditions to find liquid water on a planet's surface. And liquid water is a key ingredient in the search for life.

229. In the context of modern scientific research, consider the following statements about 'IceCube', a particle detector located at South Pole, which was recently in the news:
  1. It is the world's largest neutrino detector, encompassing a cubic kilometre of ice.
  2. It is a powerful telescope to search for dark matter.
  3. It is buried deep in the ice.

Which of the statements given above is/are correct?

Explanation: D. IceCube is an enormous particle detector located deep in the ice at the South Pole. IceCube is the world's largest neutrino detector, encompassing a cubic kilometer of ice. The IceCube telescope is a powerful tool to search for dark matter and could reveal the physical processes associated with the enigmatic origin of the highest energy particles in nature. Buried deep within the East Antarctic ice sheet at the South Pole, a giant high-energy neutrino observatory due for completion in 2009 could provide scientists, including from Europe, with an unprecedented window to the Universe, as well as a means to answer some of the most fundamental questions of astrophysics and cosmology.

230. Which one of the following was given classical language status recently?

Explanation: A. Odia (Odiya) language on 20 February 2014 became the sixth language of the country to get classical language' status after the Union Cabinet approved it. Odia is billed as the first language from the Indo-Aryan linguistic group and the case for making it a classical language was also premised on the fact that it has no resemblance to Hindi, Sanskrit, Bengali and Telugu. With this, Odia came into the same league as Sanskrit, Tamil, Telugu, Kannada and Malayalam, which have already been conferred the classical language status.

231. Indira Gandhi Prize for Peace, Disarmament and Development for 2014 was given to which one of the following?

Explanation: C. Indian space agency ISRO was conferred the Indira Gandhi Prize for Peace, Disarmament and Development for 2014 in recognition of its contribution in strengthening international cooperation in peaceful use of outer space.

232. With reference to the 'Trans-Pacific Partnership', consider the following statements:
  1. It is an agreement among all the Pacific Rim countries except China and Russia.
  2. It is a strategic alliance for the purpose of maritime security only.

Which of the statements given above is/are correct?

Explanation: D. All 12 Pacific Rim Countries have signed Trans - Pacific Partnership (TPP) agreement. These 12 countries are: Singapore, Malaysia, Vietnam, Brunei, Darussalam, Japan, Canada, United States, Mexico, Peru, Chile, Australia and New Zealand. Objectives of TPP: Lower track barriers such as tariffs; Establish a common framework for intellectual property; Enforce standards for labour low and environmental low; Establish an investor - state dispute settlement mechanism.

233. Rashtriya Garima Abhiyaan is a national campaign to

Explanation: C. It is a campaign for eradication of inhuman practice of manual scavenging and comprehensive rehabilitation of manual scavengers in India. Ashif Shaikh is well known for his role in the campaign (Rashatriya Garima Abhiyan) through various campaigns of Jan Sahas.

234. Mission Indradhanush launched by the Government of India pertains to

Explanation: A. The Government of India launched Mission Indradhanush in December 2014 with the ultimate goal to ensure that all children under the age of two years and pregnant women are fully immunized with all available vaccines.

235. With reference to Li-Fi, recently in the news, which of the following statements is/are correct?
  1. It uses light as the medium for high-speed data transmission.
  2. It is a wireless technology and is several times faster than 'Wi-Fi'.

Select the correct answer using the code given below.

Explanation: C. Both 1 and 2 statements are correct.

236. European Stability Mechanism', sometimes seen in the news, is an

Explanation: B. The European Stability Mechanism is a European Union agency that provides financial assistance, in the form of loans, to eurozone countries or as new capital to banks in difficulty.

237. The term 'Regional Comprehensive Economic Partnership' often appears in the news in the context of the affairs of a group of countries known as

Explanation: B. Regional Comprehensive Economic Partnership (RCEP) is a proposed free trade agreement (FTA) between the ten member states of the Association of Southeast Asian Nations (ASEAN) (Brunei, Burma (Myanmar), Cambodia, Indonesia, Laos, Malaysia, the Philippines, Singapore, Thailand, Vietnam) and the six states with which ASEAN has existing FTAs (Australia, China, India, Japan, South Korea and New Zealand).

238. In the context of which of the following do you sometimes find the terms 'amber box, blue box and green box' in the news?

Explanation: A. WTO Affairs.

239. Recently, which one of the following currencies has been proposed to be added to the basket of IMF's SDR?

Explanation: D. The value of the SDR or XDR of IMF is based on a basket of key international currencies reviewed every five years. In the review conducted in November 2015, the IMF decided that the Renminbi (Chinese Yuan) would be added to the basket effective October 1, 2016. From that date, the SDR or XDR basket will consist of the following five currencies: U.S. dollar 41.73%, Euro 30.93%, Chinese Yuan 10.92%, Japanese Yen 8.33%, British Pound 8.09%.

240. With reference to 'IFC Masala Bonds', sometimes seen in the news, which of the statements given below is/ are correct?
  1. The International Finance Corporation, which offers these bonds, is an arm of the World Bank.
  2. They are the rupee-denominated bonds and are a source of debt financing for the public and private sector.

Select the correct answer using the code given below.

Explanation: C. Both 1 and 2.

241. Consider the following statements:
  1. New Development Bank has been set up by APEC.
  2. The headquarters of New Development Bank is in Shanghai.

Which of the statements given above is/are correct?

Explanation: B. The New Development Bank (NDB), formerly referred to as the BRICS Development Bank, is a Multilateral Development Bank established by the BRICS states (Brazil, Russia, India, China and South Africa). Its headquarter is in Shanghai, China.

242. With reference to the International Monetary and Financial Committee (IMFC), consider the following statements:
  1. IMFC discusses matters of concern affecting the global economy, and advises the International Monetary Fund (IMF) on the direction of its work.
  2. The World Bank participates as observer in IMFC's meetings.

Which of the statements given above is/are correct?

Explanation: C. The IMFC advises and reports to the IMF Board of Governors on the supervision and management of the International Monetary and Financial System. It also considers proposals by the Executive Board to amend the Articles of Agreement and advises on any other matters that may be referred to it by the Board of Governors. A number of international institutions, including the World Bank, participate as observers in the IMFC

243. India's ranking in the 'Ease of Doing Business Index' is sometimes seen in the news. Which of the following has declared that ranking?

Explanation: C. World Bank.

244. Which of the following is/are the indicator/indicators used by IFPRI to compute the Global Hunger Index Report?
  1. Undernourishment
  2. Child stunting
  3. Child mortality

Select the correct answer using the code given below.

Explanation: C. Four Indicators are undernourished population, wasting in Under 5 children; wasting is very low weight for height i.e very thin child; Stunting in under 5 children; stunting is very low height for age i.e. very short child; Under 5 mortality rate.

245. Consider the following statements: The India-Africa Summit
  1. held in 2015 was the third such Summit
  2. was actually initiated by Jawaharlal Nehru in 1951

Which of the statements given above is/are correct?

Explanation: A. Third India-Africa Forum Summit was held in New Delhi (India) in from from 26

246. Doctors Without Borders (Medecins Sans Frontieres), often in the news, is

Explanation: B. It is a non-governmental and non-military organization established by group of French doctors in 1971 - with its Headquarter in Brussels, Belgium. They were in news during 2014-15 because of their relief work in Ebola affected countries.

247. Which of the following is not a member of 'Gulf Cooperation Council'?

Explanation: A. Iran is not the member of this middle eastern organization. Gulf Cooperation Council (GCC) is a political and economic alliance of six Middle Eastern countries

248. Belt and Road Initiative' is sometimes mentioned in the news in the context of the affairs of

Explanation: D. The Belt and Road (abbreviated B&R), One Belt, One Road (abbreviated OBOR) or the Belt and Road Initiative is a development strategy and framework, proposed by Chinese paramount leader Xi Jinping that focuses on connectivity and cooperation among countries primarily between the People

249. Consider the following pairs: Community sometimes in the affairs of mentioned in the news
  1. Kurd - Bangladesh
  2. Madhesi - Nepal
  3. Rohingya - Myanmar

Which of the pairs given above is/are correctly matched?

Explanation: C. The Kurds are an ethnic group in the Middle East Asia. The Madhesi , also referred to as Teraibasi Nepali are an indigenous ethnic group of Nepalese people who are natives of the Madhesh plains of Southern Nepal in Terai belt of South Asia. The Rohingya people, are Muslim Indo-Aryan peoples from the Rakhine State, Myanmar.

250. With reference to 'Organization for the Prohibition of Chemical Weapons (OPCW)', consider the following statements:
  1. It is an organization of European Union in working relation with NATO and WHO.
  2. It monitors chemical industry to prevent new weapons from emerging.
  3. It provides assistance and protection to States (Parties) against chemical weapons threats.

Which of the statements given above is/are correct?

Explanation: B. The OPCW Member States share the collective goal of monitoring chemical industry to prevent new weapons from re-emerging; providing assistance and protection to States Parties against chemical threats; and fostering international cooperation to strengthen implementation of the Convention and promote the peaceful use of chemistry.

251. Recently, for the first time in our country, which of the following States has declared a particular butterfly as 'State Butterfly'?

Explanation: D. In June 2015, Maharashtra government has declared the Blue Mormon (Papilio polymnestor) as the State butterfly.

252. Recently, linking of which of the following rivers was undertaken?

Explanation: B. India took a step forward in its ambitious but long-pending goal to interlink major rivers to form a national water grid. The Pattiseema project lifts flood water from the river Godavari and pumps it into the Polavaram right canal that empties into the river Krishna in Vijayawada. Andhra Pradesh Government is interlinking Krishna-Godavari rivers through the Pattiseema lift irrigation scheme.

253. Recently, which of the following States has explored the possibility of constructing an artificial inland port to be connected to sea by a long navigational channel?

Explanation: D. The Rajasthan government is exploring ways to develop an artificial inland port in Jalore by bringing in Arabian Sea water into Rajasthan through Gujarat. If implemented, Rajasthan will become accessible by water transport and will be able to shed the tag of a landlocked state. Rajasthan wants to develop artificial inland shipping port at Jalore, it'll be connected to the Arabian Sea by developing a channel along the Kutch Creek.

254. A recent movie titled 'The Man Who Knew Infinity' is based on the biography of

Explanation: A. 'The Man who knew infinite' is a biographical movie of Indian Mathematician Srinivas Ramanujan (played by Dev Patel) and his association with British mathematician GH Hardy. Ramanujan was a self-taught (autodidatic) Mathematician- born and died in Madras Presidency.

255. Consider the following statements:
  1. India has ratified the Trade Facilitation Agreement (TFA) of WTO.
  2. TFA is a part of WTO's Bali Ministerial Package of 2013.
  3. TFA came into force in January 2016.

Which of the statements given above is/are correct?

Explanation: A. TFA came into force from February 2017.

256. Broad-based Trade and Investment Agreement (BTIA)' is sometimes seen in the news in the context of negotiations held between India and

Explanation: A. India-EU BTIA negotiation has been going on for years, because of differences over data security, child labour and IPR.

257. The term 'Digital Single Market Strategy' seen in the news refers to

Explanation: C. Term was coined by EU in 2015, and was in news again in March-2016 because of Brussel Summit.

258. With reference to the 'National Intellectual Property Rights Policy', consider the following statements:
  1. It reiterates India's commitment to the Doha Development Agenda and the TRIPS Agreement.
  2. Department of Industrial Policy and Promotion is the nodal agency for regulating intellectual property rights in India.

Which of the above statements is/are correct?

Explanation: C. Both 1 and 2.

259. What is the importance of developing Chabahar Port by India?

Explanation: C. In May 2016, India and Iran signed the "historic" Chabahar port agreement, which has the potential of becoming India's gateway to Afghanistan, Central Asia and Europe.

260. The term 'Domestic Content Requirement' is sometimes seen in the news with reference to

Explanation: A. American companies contend that India's Jawaharlal Nehru Solar Mission gives preference to procurement of solar panels with Indian content. This is one type of non-tariff barrier. WTO ruled in their favour.

261. Consider the following in respect of 'National Career Service:
  1. National Career Service is an initiative of the Department of Personnel and Training, Government of India.
  2. National Career Service has been launched in a Mission Mode to improve the employment opportunities to uneducated youth of the country.

Which of the above statements is/are correct?

Explanation: B. Labour Ministry launched this portal in 2015.

262. With reference to 'National Skills Qualification Framework (NSQF)', which of the statements given below is/are correct?
  1. Under NSQF, a learner can acquire the certification for competency only through formal learning.
  2. An outcome expected from the implementation of NSQF is the mobility between vocational and general education.

Select the correct answer using the code given below:

Explanation: B. B is correct option.

263. Recognition of Prior Learning Scheme' is sometimes mentioned in the news with reference to

Explanation: A. Certifying the skills acquired by construction workers through traditional channels.

264. Which of the following statements is/are correct regarding Smart India Hackathon 2017?
  1. It is a centrally sponsored scheme for developing every city of our country into Smart Cities in a decade.
  2. It is an initiative to identify new digital technology innovations for solving the many problems faced by our country.
  3. It is a programme aimed at making all the financial transactions in our country completely digital in a decade.

Select the correct answer using the code given below:

Explanation: B. HRD Ministry's Smart India Hackathon is a 36 hours non-stop digital programming competition during which student teams will compete to offer innovative solutions for any given problem.

265. In India, it is legally mandatory for which of the following to report on cyber security incidents?
  1. Service providers
  2. Data centres
  3. Body corporate

Select the correct answer using the code given below:

Explanation: D. In the pursuance of section 70-B of the Information Technology Act, 2000 (the "IT Act"), Central Government issued the Information Technology (The Indian Computer Emergency Response Team and Manner of Performing Functions and Duties) Rules, 2013, these CERT Rules also impose an obligation on service providers, intermediaries, data centres and body corporates to report cyber incidents within a reasonable time so that CERT-In may have scope for timely action.

266. Why is a plant called Prosopis juliflora often mentioned in news?

Explanation: B. This plant is a native of Mexico, and spread throughout India. It's an aggressive colonizer, common weed of wastelands, scrublands and degraded forests. Since it's mention under the head of invasive alien species so it'd reduce biodiversity in the area.

267. Consider the following statements:
  1. The definition of "Critical Wildlife Habitat" is incorporated in the Forest Rights Act, 2006.
  2. For the first time in India, Baigas have been given Habitat Rights.
  3. Union Ministry of Environment, Forest and Climate Change officially decides and declares Habitat Rights for Primitive and Vulnerable Tribal Groups in any part of India.

Which of the statements given above is/are correct?

Explanation: A. Baigas are considered as a particularly vulnerable tribal group (PVTG) in the Indian Constitution and rely mostly on shifting cultivation, forest produce and fishing for sustenance, spread over forested areas of Madhya Pradesh and Chhattisgarh. In 2016, they became India's first community to get habitat rights. Under FRA Act, district level Committees have to adjudicate the rights, whereas Tribal ministry only issues guidelines.

268. With reference to organic farming in India, consider the following statements:
  1. The National Programme for Organic Production' (NPOP) is operated under the guidelines and directions of the Union Ministry of Rural Development.
  2. The Agricultural and Processed Food Products Export Development Authority' (APEDA) functions as the Secretariat for the implementation of NPOP.
  3. Sikkim has become India's first fully organic State.

Which of the statements given above is/are correct?

Explanation: B. This scheme is under Commerce ministry, but Secretariat assistance by APEDA. Sikkim indeed India's first fully organic state.

269. Which of the following is/are the aim/aims of "Digital India" Plan of the Government of India?
  1. Formation of India's own Internet companies like China did
  2. Establish a policy framework to encourage overseas multinational corporations that collect Big Data to build their large data centres within our national geographical boundaries.
  3. Connect many of our villages to the Internet and bring Wi-Fi to many of our school, public places and major tourist

Select the correct answer using the code given below

Explanation: B. Digital India programme has three vision areas namely, digital infrastructure as a utility for every citizen, governance and services on demand and digital empowerment of citizens by bridging the digital divide in the country.

270. With reference to Pradhan Mantri Kaushal Vikas Yojana, consider the following statements
  1. It is the flagship scheme of the Ministry of Labour and Employment.
  2. It, among other things, will also impart training in soft skills, entrepreneurship, financial and digital literacy.
  3. It aims to align the competencies of the unregulated workforce of the country to the National Skill Qualification Framework.

Which of the statements given above is/are correct?

Explanation: C. PMKVY is the flagship outcome-based skill training scheme of the Ministry of Skill Development and Entrepreneurship (MSDE). Ministry of Skill Development is created with core focus on converging all skill development initiatives in the country under one National Skills Qualification Framework (NSQF).

271. International Labour Organization's Conventions 138 and 182 are related to

Explanation: A. ILO Convention No. 138: minimum age of entry into work shall not be less than the age of completion of compulsory schooling and, in any case, shall not be less than 15 years. ILO Convention No. 182: "Worst Forms of Child Labour" have to be prohibited urgently.

272. In the Indian context, what is the implication of ratifying the 'Additional Protocol' with the 'International Atomic Energy Agency (IAEA)'?

Explanation: A. Under the old IAEA safeguards, all NPT signatories would specify their nuclear sites and IAEA would carry out inspections in the specified sites. Thus, IAEA, under the old safeguards, could only carry out inspection for unauthorized activities only at designated or specified sites declared by a country. This basically left an option open for states to carry out covert nuclear programmes - as it happened in case of Iraq. Thus, in 1993, the IAEA designed Additional Protocols (AP) to tighten the existing safeguarding regime. However, India specific Additional Protocols (AP) do not give IAEA the right to hinder or interfere with activities which are outside the scope of India's safeguard agreements, thus recognizing that India reserves a right to a military nuclear program outside IAEA agreement.

273. What is/are the consequence/consequences of a country becoming the member of the Nuclear Suppliers Group'?
  1. It will have access to the latest and most efficient nuclear technologies.
  2. It automatically becomes a member of "The Treaty on the Non-Proliferation of Nuclear Weapons (NPT)".

Which of the statements given above is/are correct?

Explanation: A. By joining NSG, a country can access to sophisticated nuclear technology. India wishes to be a part of the NSG.

274. Rule of Law Index is released by which of the following?

Explanation: D. World Justice Project

275. Consider the following pairs of Regions sometimes mentioned in news vs Country:
  1. Catalonia : Spain
  2. Crimea : Hungary
  3. Mindanao : Philippines
  4. Oromia : Nigeria

Which of the pairs given above are correctly matched?

Explanation: C. Catalonia had been in news due to referendum where people decided to vote for secession from Spain. Crimea had been gifted to Ukraine in 1954 but Putin annexed the province back in 2014. In 2017, India has decided to give 25 Million Pesos aid to Philippines to fight Islamic State in Mindanao Province. This is the first time that India has decided to extend monetary assistance outside to any state to fight terrorism. This signals a rise of India's image as a Net Security Provider. Oromia is not in Nigeria but in Ethiopia, it has been in news in 2017, due to Ethnic violence, terrorism and general anarchy that is common in such African countries and also in some Indian states.

276. Very recently, in which of the following countries have lakhs of people either suffered from severe famine/ acute malnutrition or died due to starvation caused by war/ethnic conflicts?

Explanation: D. Yemen and South Sudan.

277. The term "two-state solution" is sometimes mentioned in the news in the context of the affairs of

Explanation: B. Palestine under Israeli control to advocating the two states solution. It wants the creation of Arab Palestine state in the Gaza strip and West Bank, along side the state of Israel.

278. Consider the following pairs Town sometime mentioned in news: Country:
  1. Aleppo : Syria
  2. Kirkuk : Yemen
  3. Mosul : Palestine
  4. Mazar -i- Sharif: Afghanistan

Which of the pairs given above are correctly matched?

Explanation: B. Correct option is B.

279. What is "Terminal High Altitude Area Defense (THAAD)", sometimes seen in the news?

Explanation: C. The THAAD system is designed to intercept and destroy short and medium-range ballistic missiles during their final phase of flight. USA installed this system in South Korea in 2017, in the wake North Korean missile tests. But this angered China, who feels that deployment of American armaments will upset the regional security balance.

280. The Global Competitiveness Report is published by the

Explanation: C. The Global Competitiveness Report (GCR) is a yearly report published by the World Economic Forum.

281. Atal Innovation Mission is set up under the

Explanation: C. AlM's objectives are to create and promote an ecosystem of innovation and entrepreneurship across the country at school, university, research institutions, MSME and industry levels. The Mission has been set up under NITI Aayog.

282. Recently, there was a growing awareness in our country about the importance of Himalayan nettle (Girardinia diversifolia) because it is found to be a sustainable source of

Explanation: D. Himalayan nettle(Girardinia diversifolia) is found to be a sustainable source of textile fiber. The fabric and the things made from it. For the Himalayan Indian Nettle, Government of india has granted approval for a project titled Development of products from Himalayan Indian nettle to Department of Jute and Fibre Technology, Institute of Jute Technology, University of Kolkata.

283. Among the following, which one is the largest exporter of rice in the world in the last five years?

Explanation: B. India has been the world's top rice exporter since the beginning of this decade. India emerged the world's largest rice exporter in 2011-12, displacing Thailand from its leadership position.

284. In India, the use of carbofuran, methyl parathion, phorate and triazophos is viewed with apprehension. These chemicals are used as

Explanation: A. Carbofuran, Phorate and Triazophos are pesticides used in agriculture.

285. Consider the following statements about Particularly Vulnerable Tribal Groups (PVTGs) in India:
  1. PVTGs reside in 18 States and one Union Territory.
  2. A stagnant or declining population is one of the criteria for determining PVTG status.
  3. There are 95 PVTGs officially notified in the country so far.
  4. Irular and Konda Reddi tribes are included in the list of PVTGs.

Which of the statements given above are correct?

Explanation: C. 1, 2 and 4.

286. Consider the following statements:
  1. The United Nations Convention against Corruption (UNCAC) has a 'Protocol against the Smuggling of Migrants by Land, Sea and Air'.
  2. The UNCAC is the ever-first legally binding global anti-corruption instrument.
  3. A highlight of the United Nations Convention against Transnational Organized Crime (UNTOC) is the inclusion of a specific chapter aimed at returning assets to their rightful owners from whom they had been taken illicitly.
  4. The United Nations Office on Drugs and Crime (UNODC) is mandated by its member States to assist in the implementation of both UNCAC and UNTOC.

Which of the statements given above are correct?

Explanation: C. 2 and 4 only.

287. Consider the following statements:
  1. As per recent amendment to the Indian Forest Act, 1927, forest dwellers have the right to fell the bamboos grown on forest areas.
  2. As per the Scheduled Tribes and Other Traditional Forest Dwellers (Recognition of Forest Rights) Act, 2006, bamboo is a minor forest produce.
  3. The Scheduled Tribes and Other Traditional Forest Dwellers (Recognition of Forest Rights) Act, 2006 allows ownership of minor forest produce to forest dwellers.

Which of the statements given above is/are correct?

Explanation: B. 2 and 3 only.

288. Consider the following statements: As per the Industrial Employment (Standing Orders) Central (Amendment) Rules, 2018
  1. if rules for fixed-term employment are implemented, it becomes easier for the firms/companies to lay off workers
  2. no notice of termination of employment shall be necessary in the case of temporary workman

Which of the statements given above is/are correct?

Explanation: C. Both 1 and 2.

289. With reference to the management of minor minerals in India, consider the following statements:
  1. Sand is a 'minor mineral' according to the prevailing law in the country.
  2. State Governments have the power to grant mining leases of minor minerals, but the powers regarding the formation of rules related to the grant of minor minerals lie with the Central Government.
  3. Stale Governments have the power to frame rules to prevent illegal mining of minor minerals.

Which of the statements given above is/are correct?

Explanation: A. 1 and 3 only.

290. Which of the following statements is/are correct regarding the Maternity Benefit (Amendment) Act, 2017?
  1. Pregnant women are entitled for three months pre-delivery and three months postdelivery paid leave.
  2. Enterprises with creches must allow the mother minimum six creche visits daily.
  3. Women with two children get reduced entitlements.

Select the correct answer using the code given below.

Explanation: C. This new bill amends the Maternity Benefit Act of 1961. Under the new provision, women are entitled to visit creches four times a day. Maternity benefit of 26 weeks are provided. Out of 26 weeks, Up to eight weeks can be claimed before delivery. However, one can instead take the entire 26 weeks of leave after the delivery. If the woman has more than two surviving children, the maternity benefit is for 12 weeks only.

291. Which one of the following is not a sub-index of the World Bank's 'Ease of Doing Business Index'?

Explanation: A. Sub Index of Ease of Doing Buisness Index comprises starting a buisness,Getting a location (labour market regulation, constuction permits, getting electrictiy, registering property), Accessing finance (Getting credit, Protecting Minority investor), Dealing with day to day operation(Paying taxes, enforcing contracts).

292. Which of the following adopted a law on data protection and privacy for its citizens known as General Data Protection Regulation

Explanation: C. The European Union.

293. Recently, India signed a deal known as

Explanation: B. During the visit of H. E. Vladimir Putin, President of the Russian Federation to India, Action Plan for Prioritization and Implementation of Co-operation Areas in the Nuclear Field Identified Jointly by India and Russia was signed on 5th October, 2018 in New Delhi.

294. The word 'Denisovan' is sometimes mentioned in media in reference to

Explanation: B. For the first time, scientists have found fossils from an extinct ancient human lineage known as the Denisovans outside of Siberia. Denisovans were an extinct group of hominins that were close relatives of Neanderthals. They are known primarily from a handful of fossil fragments found at Denisova Cave in Siberia, and from genetic clues that linger in the DNA of people across Asia.

Question No.
Toppers

Top Popular Courses


Newsletter Subscription
SMS Alerts

Important Links

UPSC GS Mains Crash Course - RAW Prelims Answer Key 2024